Quantcast
Channel: L’esperto risponde - Matematica » limiti
Browsing all 10 articles
Browse latest View live

Un limite

Ricevo da Mari il seguente quesito: Calcola il seguente limite: \[\underset{x\to 0}{\mathop{\lim }}\,\frac{1-\cos \frac{{{x}^{2}}}{x+1}}{{{\ln }^{2}}\left( \frac{{{x}^{2}}+x+1}{x+1}...

View Article



Una funzione invertibile

Ricevo da Jessica il seguente problema: a) Determina il campo di esistenza della funzione \[f\left( x \right)=\frac{\ln x}{1-2\ln x}\] e calcola i limiti per \(x\to {{0}^{+}}\) e per \(x\to +\infty\)....

View Article

Uno studio di funzione

Ricevo da Rosa la seguente funzione da studiare: \[y=\frac{{{x}^{2}}+9}{x-1}\quad .\]

View Article

Sul teorema di de l’Hopital nel calcolo di limiti

Ricevo da Mari una domanda su un limite del tipo rapporto di infiniti simultanei in relazione al teorema di de l'Hopital.

View Article

Sul teorema di Lagrange

Ricevo da Claudia la seguente domanda: Considera la funzione \(f(x)\) cosi composta: \[f\left( x \right):\quad \frac{3x-2a}{x+b}\quad se\quad 0\le x\le 1\] \(f\left( x\right):\quad {e}^{x-1}\quad\) se 1

View Article


Uno studio di funzione

Ricevo da Asia la richiesta di studiare la seguente funzione: \[f\left( x \right)=\ln \left| \cos x \right|\quad .\]

View Article

Uno studio di funzione

Ricevo da Asia la seguente funzione da studiare: \[f\left( x \right)=\ln \left| \arctan x \right|\quad .\]

View Article

Un limite e il teorema di de l’Hopital

Ricevo da Beatrice il seguente quesito: Calcola il seguente limite applicando, qualora sia possibile, il teorema di De l’Hospital: \[\underset{x\to +\infty }{\mathop{\lim...

View Article


Un integrale improprio

Ricevo da Francesca il seguente integrale improprio: \[\int\limits_{-1}^{+\infty }{\frac{1}{\left( x+2 \right)\sqrt{\left| x \right|}}}dx\quad .\]

View Article


Una funzione logaritmica

Ricevo da Asia la seguente funzione da studiare: \[f\left( x \right)=\ln \left( {{x}^{2}}+1 \right)-1\quad .\]

View Article
Browsing all 10 articles
Browse latest View live